- PowerScore Staff
- Posts: 5972
- Joined: Mar 25, 2011
- Thu Jan 21, 2016 12:00 am
#46120
Complete Question Explanation
(The complete setup for this game can be found here: lsat/viewtopic.php?t=6424)
The correct answer choice is (B)
The question stem stipulates that exactly four applicants are hired, which means that at least four applicants are interviewed:
From the contrapositive of the last rule, when O is not hired, then M is not hired or L is not interviewed (or hired, then). But, consider what would happen if L is not interviewed: from the contrapositives of the first two rules, J could not be interviewed, and G could not be interviewed. That would mean that O could not be hired, L could not be hired, J could not be hired, and G could not be hired, leaving only three applicants available to be hired, violating the stipulation in this question that exactly four people are hired. Thus, L must be interviewed, and, correspondingly (to meet the contrapositive of the sixth rule) M cannot be hired. Thus answer choice (B), M is hired, cannot be true and is the correct answer.
(The complete setup for this game can be found here: lsat/viewtopic.php?t=6424)
The correct answer choice is (B)
The question stem stipulates that exactly four applicants are hired, which means that at least four applicants are interviewed:
From the contrapositive of the last rule, when O is not hired, then M is not hired or L is not interviewed (or hired, then). But, consider what would happen if L is not interviewed: from the contrapositives of the first two rules, J could not be interviewed, and G could not be interviewed. That would mean that O could not be hired, L could not be hired, J could not be hired, and G could not be hired, leaving only three applicants available to be hired, violating the stipulation in this question that exactly four people are hired. Thus, L must be interviewed, and, correspondingly (to meet the contrapositive of the sixth rule) M cannot be hired. Thus answer choice (B), M is hired, cannot be true and is the correct answer.
You do not have the required permissions to view the files attached to this post.
Dave Killoran
PowerScore Test Preparation
Follow me on X/Twitter at http://twitter.com/DaveKilloran
My LSAT Articles: http://blog.powerscore.com/lsat/author/dave-killoran
PowerScore Podcast: http://www.powerscore.com/lsat/podcast/
PowerScore Test Preparation
Follow me on X/Twitter at http://twitter.com/DaveKilloran
My LSAT Articles: http://blog.powerscore.com/lsat/author/dave-killoran
PowerScore Podcast: http://www.powerscore.com/lsat/podcast/